GMAT Club
close
Generic [Bot] online
My chats
Join group
New group
New private
Archived groups

GMAT Verbal Chat

GMAT Quantitative Chat

GMAT Data Insight Chat

MBA Chat

go back
Verbal Question of the Day 4 members online out of 123577
dots
Forum discussion
Chat members
User avatar
siddharth_ wrote:
Hi Can anyone please help me with the following doubt -> I just started preparing 2 weeks ago using e-gmat’s subscribed course. I have a question regarding CR practice. Currently I am spending 8 - 12 mins in solving the 650-720 level questions (level stated by e-gmat in their practice files). Although I got 8/9 correct, but the timing seems like a serious issue to me. Should I be worried at this stage?

Im not a pro , just started my prep as well. But 8-12 mins per questions of CR u mean ?
chevron down
Quote
Private
Delete
Pin
User avatar
kumar.avanish wrote:
Im not a pro , just started my prep as well. But 8-12 mins per questions of CR u mean ?

yes
chevron down
Quote
Private
Delete
Pin
User avatar
siddharth_ wrote:
yes

Well. In actual GMAT . U get around 2 min max for each questions . So decide yourself. Better to judge which questions u can do or guess the answer mark and get rid of , if its there to just waste the time.
chevron down
Quote
Private
Delete
Pin
User avatar
kumar.avanish wrote:
Im not a pro , just started my prep as well. But 8-12 mins per questions of CR u mean ?

yes
chevron down
Quote
Private
Delete
Pin
User avatar


111.5.png

chevron down
Quote
Private
Delete
Pin
User avatar
please explain this question. IT’S AN OFFICIAL QUESTION
chevron down
Quote
Private
Delete
Pin
User avatar
I guess this is a very new question. Don’t see anything on the thread. That said, here are my quick 2-cents:
-Claim: The data of the last 50 years is proof that it is unlikely that the cost of treating any particular disease will reduce just because of med technological improvements
-Logic: Because… while med tech has improved tremendously, annual overall spending on disease treatment has not reduced, it has also risen sharply
-One flaw I can think of: The cost of treating any particular disease may really have come down. But because of more number of patients, the annual overall spend increased rather than decreased. There are more such.
-Question: Flaw EXCEPT. Find the one thing which, even if it possibly changes, does not make the argument vulnerable.
-Choice A: if the average age changes, say increases, more elders could mean more patients than before – as an example. Then, even if the technology is better, the overall spend on disease treatment could still increase. Average age could tangibly matter here.
-Choice B: Due to inflation, the same treatment costs more. A treatment that cost 1L Rs may now cost 3L Rs. Thus, despite the technology getting better, the overall spend on disease treatment could still increase.
-Choice C: Greater size of population could straight-up mean more patients. Thus, despite the technology getting better, the overall spend on disease treatment could still increase.
-Choice D: Irrespective of number of medical researchers, we already know that med tech has improved tremendously. What difference does it make whether this improvement came from an unchanged number of medical researchers, or whether this number presumably increased? This choice seems to be the best candidate.
-Choice E: A change in type of disease can clearly impact the overall spend on disease treatment. For instance, despite med tech improvement, a huge chunk of expenses are getting incurred now due to a greater contribution of expensive diseases in the population (say, lot more dialysis or cancer cases). Then, even if the technology is better, the overall spend on disease treatment could still increase
-Choice D looks like the best candidate.

Hope this helps!
chevron down
Quote
Private
Delete
Pin
User avatar
How much time should one take to solve a question like this one?

Screenshot 2024-04-14 at 1.46.38 PM.png

chevron down
Quote
Private
Delete
Pin
User avatar
siddharth_ wrote:
How much time should one take to solve a question like this one?

Is the answer D?
chevron down
Quote
Private
Delete
Pin
User avatar
I believe the answer is D.
chevron down
Quote
Private
Delete
Pin
User avatar
Correct. Can you please tell me the time that it took you to solve this one?
chevron down
Quote
Private
Delete
Pin
User avatar
I have solved this one before. Just under 2 min then from what I remember.
chevron down
Quote
Private
Delete
Pin
User avatar
siddharth_ wrote:
Correct. Can you please tell me the time that it took you to solve this one?

Yeah around 2 mins. Definitely under 2 mins 20 secs.
The evaluation of options could take some time although option D stood out clearly so it was easier to choose this as the answer.
chevron down
Quote
Private
Delete
Pin
User avatar
But if you have just started practicing, I think you should focus more on why an answer is correct and why others are incorrect. Focus on timing at the start could be counter productive.
chevron down
Quote
Private
Delete
Pin
User avatar
Agreed with @Nullbyte!
chevron down
Quote
Private
Delete
Pin
User avatar
I also believe D is the answer
chevron down
Quote
Private
Delete
Pin
User avatar
Get ready for a power practice session https://gmatclub.com/forum/redirect.php?id=le4ubis2ro

AP14-SNDQZ-MX650.jpg

chevron down
Quote
Private
Delete
Pin
User avatar
Hi all, I am new to the group, can someone tell me when and how do we get to know these quiz schedules?
chevron down
Quote
Private
Delete
Pin
User avatar
Hi and welcome! We usually usually run them once a week on Sundays. Maybe we should start a poll for voting on the topic that we should run? 🤷‍♂️
chevron down
Quote
Private
Delete
Pin
User avatar
Here you go: https://gmatclub.com/forum/weekly-gmat- ... 28565.html - feel free to suggest a topic here (anything specific or general).
chevron down
Quote
Private
Delete
Pin
User avatar
Critical Reasoning Butler: April 2024
April 15CR 1CR 2
­
chevron down
Quote
Private
Delete
Pin
User avatar
Bunuel wrote:
Critical Reasoning Butler: April 2024
April 15CR 1CR 2
­

In CR 1 , A is the ans ?
chevron down
Quote
Private
Delete
Pin
User avatar
What about option b
chevron down
Quote
Private
Delete
Pin
User avatar
CR2 E
chevron down
Quote
Private
Delete
Pin
User avatar
April 16
CR 1
CR 2
chevron down
Quote
Private
Delete
Pin
User avatar
Critical Reasoning Butler: April 2024
April 16CR 1CR 2
­
chevron down
Quote
Private
Delete
Pin
User avatar
Critical Reasoning Butler: April 2024
April 17CR 1CR 2
­
chevron down
Quote
Private
Delete
Pin
User avatar
CR 1 APR 17 D is the ans?
chevron down
Quote
Private
Delete
Pin
User avatar
Four neighbouring countries produce a third of a world’s harvest of sugarcane. During the last decade, Vernia’s sugarcane output was greater than that of Browland. However, because Sylvania had a greater output of sugarcane than Borodia’s, Vernia must have produced more sugarcane than Borodia.

If taken into consideration, any of the following makes the argument above logically correct EXCEPT.

A) Browland’s output sugarcane was greater than Borodia

B) Vernia and Sylvania produced the same amount of sugarcane

C) Vernia’s sugarcane output was less than that of Sylvania

D) Browland and Sylvania produced the same amount of sugarcane

E) Sylvania produced less sugarcane than Browland


Help please..
chevron down
Quote
Private
Delete
Pin
User avatar
I think C
chevron down
Quote
Private
Delete
Pin
User avatar
Grinreaper4334 wrote:
I think C

Yes
can you provide an explanation.
I made a mistake and don’t know where
chevron down
Quote
Private
Delete
Pin
User avatar
The question asks which of the statements furthers the conclusion given in the question. In C , if Vernia’s output is less than Sylvania, then it is possible that Vernia could produce less output than Borodia as we are not given any evidence of Vernia’s output being more than Borodia’s
chevron down
Quote
Private
Delete
Pin
User avatar
Edoua wrote:
Four neighbouring countries produce a third of a world’s harvest of sugarcane. During the last decade, Vernia’s sugarcane output was greater than that of Browland. However, because Sylvania had a greater output of sugarcane than Borodia’s, Vernia must have produced more sugarcane than Borodia. If taken into consideration, any of the following makes the argument above logically correct EXCEPT. A) Browland’s output sugarcane was greater than Borodia B) Vernia and Sylvania produced the same amount of sugarcane C) Vernia’s sugarcane output was less than that of Sylvania D) Browland and Sylvania produced the same amount of sugarcane E) Sylvania produced less sugarcane than Browland Help please..

Here is the link to this question: https://gmatclub.com/forum/four-neighbo ... l#p3385582

I added my explanation. Hope it helps.
chevron down
Quote
Private
Delete
Pin
User avatar


Screenshot 2024-04-18 104817.png

chevron down
Quote
Private
Delete
Pin
User avatar
I think the answer is B
chevron down
Quote
Private
Delete
Pin
User avatar
I think its E
chevron down
Quote
Private
Delete
Pin
User avatar
Whats the ans?
chevron down
Quote
Private
Delete
Pin
User avatar
reason for eliminating A
chevron down
Quote
Private
Delete
Pin
User avatar
HarshR9 wrote:
Here is the link to this question: https://gmatclub.com/forum/four-neighbo ... l#p3385582 I added my explanation. Hope it helps.

Thanks
chevron down
Quote
Private
Delete
Pin
User avatar
Yup E it is
chevron down
Quote
Private
Delete
Pin
User avatar
I think B
chevron down
Quote
Private
Delete
Pin
User avatar
Aishna1034 wrote:
reason for eliminating A

A is actually a weakener. People are saying that because of relaxed laws, maybe more dangerous fire works are being sold and that’s why more burns.
A is saying that the new fireworks don’t include powerful fireworks. So the relaxation probably didn’t have anything to do with more number of severe burns
chevron down
Quote
Private
Delete
Pin
User avatar
In fact A, B and C all are weakeners I think
chevron down
Quote
Private
Delete
Pin
User avatar
I think the answer is E.
chevron down
Quote
Private
Delete
Pin
User avatar
Edoua wrote:
Four neighbouring countries produce a third of a world’s harvest of sugarcane. During the last decade, Vernia’s sugarcane output was greater than that of Browland. However, because Sylvania had a greater output of sugarcane than Borodia’s, Vernia must have produced more sugarcane than Borodia. If taken into consideration, any of the following makes the argument above logically correct EXCEPT. A) Browland’s output sugarcane was greater than Borodia B) Vernia and Sylvania produced the same amount of sugarcane C) Vernia’s sugarcane output was less than that of Sylvania D) Browland and Sylvania produced the same amount of sugarcane E) Sylvania produced less sugarcane than Browland Help please..

This question discussed here: https://gmatclub.com/forum/four-neighbo ... 08791.html
chevron down
Quote
Private
Delete
Pin
User avatar
Aishna1034 wrote:
Yup E it is

Can you please tell me the source of that question? Thank you!
chevron down
Quote
Private
Delete
Pin
User avatar
Critical Reasoning Butler: April 2024
April 18CR 1CR 2
­
chevron down
Quote
Private
Delete
Pin
User avatar
Hi, I wanted to go through some RCs on the mobile app of GMAT club...
But I don’t see the option to hide timer statistics...
Is it the same with everyone?
chevron down
Quote
Private
Delete
Pin
User avatar
Critical Reasoning Butler: April 2024
April 19CR 1CR 2
­
chevron down
Quote
Private
Delete
Pin
User avatar
I am getting a 93 - 95 %ile in E-GMAT’s Critical thinking ability quizzes
chevron down
Quote
Private
Delete
Pin
User avatar
how accurate are they?
chevron down
Quote
Private
Delete
Pin
User avatar
idk much about it but may be they are precise.
chevron down
Quote
Private
Delete
Pin
User avatar
does anyone has experience about egmat’s ability quiz and actual test level
chevron down
Quote
Private
Delete
Pin
User avatar
E-GMAT’s Critical thinking ability quizzes
chevron down
Quote
Private
Delete
Pin
User avatar
where are u giving these ?
chevron down
Quote
Private
Delete
Pin
User avatar
in their verbal scholaranium
chevron down
Quote
Private
Delete
Pin
User avatar
there are 3 types of quizzes
chevron down
Quote
Private
Delete
Pin
User avatar
Where can I get some free DI material to practice, I am done with official guide, Official DI and MBA.com questions? Any recommendations?
chevron down
Quote
Private
Delete
Pin
User avatar
Cementing , Ability and custom
chevron down
Quote
Private
Delete
Pin
User avatar
The demand for Korean horror films in America will fall considerably since American horror films are back in demand there, impacting the overall demand for Korean films. However, the American viewers’ demand for Korean films will increase again. Two years back when the American romance film genre went through a revival in popularity in America, the demand for Korean romantic films in America went down; however, the demand for Korean film has returned to its previous level since then.


Which of the following, if true, best supports the assertion that the demand for Korean films may not return to its previous level?

A) In the past two years some very successful American Horror films were actually based on successful Korean Horror films.

B) The Korean horror films were significantly responsible for contributing to the revival in demand for Korean films after it dropped two years back.

C) Various surveys show that the movie business in America is almost at its peak and is likely to go down after it booms in the next one year.

D) The demand for Korean romantic films has not decreased in the last two years.

E) American sci-fi films have always been more popular in America than Korean or America horror films have been.


Helps me to solve please
chevron down
Quote
Private
Delete
Pin
User avatar
The answer is B
chevron down
Quote
Private
Delete
Pin
User avatar
Edoua wrote:
The demand for Korean horror films in America will fall considerably since American horror films are back in demand there, impacting the overall demand for Korean films. However, the American viewers’ demand for Korean films will increase again. Two years back when the American romance film genre went through a revival in popularity in America, the demand for Korean romantic films in America went down; however, the demand for Korean film has returned to its previous level since then.


Which of the following, if true, best supports the assertion that the demand for Korean films may not return to its previous level?

A) In the past two years some very successful American Horror films were actually based on successful Korean Horror films.

B) The Korean horror films were significantly responsible for contributing to the revival in demand for Korean films after it dropped two years back.

C) Various surveys show that the movie business in America is almost at its peak and is likely to go down after it booms in the next one year.

D) The demand for Korean romantic films has not decreased in the last two years.

E) American sci-fi films have always been more popular in America than Korean or America horror films have been.


Helps me to solve please

­Check here:

https://gmatclub.com/forum/the-demand-f ... 06939.html
chevron down
Quote
Private
Delete
Pin
User avatar
chevron down
Quote
Private
Delete
Pin
User avatar
karanj179 wrote:
Where can I get some free DI material to practice, I am done with official guide, Official DI and MBA.com questions? Any recommendations?

you can check here. https://gmatclub.com/forum/data-insight ... -1280.html
chevron down
Quote
Private
Delete
Pin
User avatar
Critical Reasoning Butler: April 2024
April 22CR 1CR 2
­
chevron down
Quote
Private
Delete
Pin
User avatar
Critical Reasoning Butler: April 2024
April 23CR 1CR 2
­
chevron down
Quote
Private
Delete
Pin
User avatar
Consumer advocate: There is no doubt that the government is responsible for the increased cost of gasoline, because the government’s policies have significantly increased consumer demand for fuel, and as a result of increasing demand, the price of gasoline has risen steadily.

Which one of the following is an assumption required by the consumer advocate’s argument?


(A) The government can bear responsibility for that which it indirectly causes.

(B) The government is responsible for some unforeseen consequences of its policies.

(C) Consumer demand for gasoline cannot increase without causing gasoline prices to increase.

(D) The government has an obligation to ensure that demand for fuel does not increase excessively.

(E) If the government pursues policies that do not increase the demand for fuel, gasoline prices tend to remain stable
chevron down
Quote
Private
Delete
Pin
User avatar
Anyone wna try this question?
chevron down
Quote
Private
Delete
Pin
User avatar
Official LSAT question
chevron down
Quote
Private
Delete
Pin
User avatar
C seems correct I guess
chevron down
Quote
Private
Delete
Pin
User avatar
malhar.mohite wrote:
C seems correct I guess

C is already stated in the argument.
chevron down
Quote
Private
Delete
Pin
User avatar
Nullbyte wrote:
Anyone wna try this question?

Is it A ?
chevron down
Quote
Private
Delete
Pin
User avatar
SATYAM7777 wrote:
Is it A ?

Yup
chevron down
Quote
Private
Delete
Pin
User avatar
SATYAM7777 wrote:
C is already stated in the argument.

So why do you think C is wrong? Do you mean that we have to assume the premise to be true? And what’s given in C is actually a premise?
chevron down
Quote
Private
Delete
Pin
User avatar
Assumption must be hideen
chevron down
Quote
Private
Delete
Pin
User avatar
hidden
chevron down
Quote
Private
Delete
Pin
User avatar
What do you mean?
chevron down
Quote
Private
Delete
Pin
User avatar
demand increase -> gasoline price increase not equal to gasoline price increase is necessary condition
chevron down
Quote
Private
Delete
Pin
User avatar
Hi all, GMAT FOCUS still has Reading Comprehension questions right?
chevron down
Quote
Private
Delete
Pin
User avatar
studywithpammy wrote:
Hi all, GMAT FOCUS still has Reading Comprehension questions right?

_________________________________
­Right.
chevron down
Quote
Private
Delete
Pin
User avatar
SATYAM7777 wrote:
C is already stated in the argument.

I wouldn’t say it’s stated in the argument but rather assumed for the purpose of the statement
chevron down
Quote
Private
Delete
Pin
User avatar
But I agree that A sounds more correct
chevron down
Quote
Private
Delete
Pin
User avatar
Critical Reasoning Butler: April 2024
April 24CR 1CR 2
chevron down
Quote
Private
Delete
Pin
User avatar
guys i need help in increasing accuracy in verbal . Any suggestions
chevron down
Quote
Private
Delete
Pin
User avatar
HarishD wrote:
Hi Bunuel, Should we expect such questions in the GMAT ???

Those are for practice and similar to GMAT questions
chevron down
Quote
Private
Delete
Pin
User avatar
jurgenklopp wrote:
guys i need help in increasing accuracy in verbal . Any suggestions

­Read super carefully what is written in an RC or CR passage.

The key is to understand. Even a single word can make the difference
chevron down
Quote
Private
Delete
Pin
User avatar
Critical Reasoning Butler: April 2024
April 25CR 1CR 2
­
chevron down
Quote
Private
Delete
Pin
User avatar
got it
chevron down
Quote
Private
Delete
Pin
User avatar
carcass wrote:
­Read super carefully what is written in an RC or CR passage. The key is to understand. Even a single word can make the difference

apart from og from where should i practice qustionsin gmatclub
chevron down
Quote
Private
Delete
Pin
User avatar
jurgenklopp wrote:
apart from og from where should i practice qustionsin gmatclub

where can i access og question pls help
chevron down
Quote
Private
Delete
Pin
User avatar
Critical Reasoning Butler: April 2024
April 26CR 1CR 2
chevron down
Quote
Private
Delete
Pin
User avatar
Where can I find RC questions?
chevron down
Quote
Private
Delete
Pin
User avatar
ML200211 wrote:
Where can I find RC questions?

Maybe, if it helps https://gmatclub.com/forum/reading-comp ... on-rc-137/
chevron down
Quote
Private
Delete
Pin
User avatar
jurgenklopp wrote:
apart from og from where should i practice qustionsin gmatclub

rajat163, ML200211, you can select OG and RC questions there

Screenshot 2024-04-26 174316.png

chevron down
Quote
Private
Delete
Pin
User avatar
winterschool wrote:
rajat163, ML200211, you can select OG and RC questions there

I think that I have tried to help them by sharing the right RC sub-forum link. Thank you!
chevron down
Quote
Private
Delete
Pin
User avatar
poojaarora1818 wrote:
I think that I have tried to help them by sharing the right RC sub-forum link. Thank you!

I appreciate that.
chevron down
Quote
Private
Delete
Pin
User avatar
winterschool wrote:
I appreciate that.

Thank you for your support and guidance as always. I will continue to serve club in future as much I can. As I have learnt so many things from the Gmat Club. Thanks!
chevron down
Quote
Private
Delete
Pin